Which one of the following, if substituted for the condition that Lewis and Ota do not lecture on the same day as eac...

ankita96 on August 8, 2020

Ans choice D

Hi, i read your explanation in the previous thread. I used the same logic, then why is D wrong. J and k have to be together but j and m must also be on separate days right? Thanks in advance!

Replies
Create a free account to read and take part in forum discussions.

Already have an account? log in

ankita96 on August 8, 2020

I would also like to know why E would be wrong too

Skylar on August 9, 2020

@ankita96, happy to help!

We are looking to replace the existing rule that L and O cannot go on the same day (Rule #3).

(A) "J and K must go on the same day" is correct. When this is the case, we have M and N on one day (Rule #2) and J and K on the other. This leaves only one remaining spot open on each day, one for L and one for O. Therefore, L and O will never be on the same day as each other.

(D) "J and M cannot go on the same day" is incorrect. All we know from this is that M and N go on one day (Rule #2) and J is on the other. We are never told that J and K must go together here. It would still be possible to have L and O together in the following scenario, so (D) is not an accurate rule replacement:
1: J M
2: L N
3: O K
T F

(E) "K and N cannot go on the same day" is incorrect. Similar to (D), all we know from this is that M and N go on one day (Rule #2) and K is on the other. It would still be possible to have L and O together in the following scenario, so (E) is not an accurate rule replacement:
1: J L
2: M O
3: N K
T F

Does that make sense? Hope it helps! Please let us know if you have any other questions.

ankita96 on August 10, 2020

oh right! I pictured it wrong. Thank you, this was really helpful.